Jump to content

Photo

Topic tổng hợp một số bất đẳng thức trong kì thi MO các nước

* * * * * 16 votes

  • Please log in to reply
501 replies to this topic

#301
Nguyen Minh Hai

Nguyen Minh Hai

    Thiếu úy

  • Thành viên
  • 666 posts

Bài 111(Greece MO)(Nguyễn Duy Khương): Cho a,b,c là các số thực dương sao cho ab+bc+ca=1 . Cmr:

 

                           

                          $\sqrt{\frac{a^2+1}{b^2+6bc+c^2}}+\sqrt{\frac{b^2+1}{c^2+6ca+a^2}}+\sqrt{\frac{c^2+1}{a^2+6ab+b^2}}\geq \frac{3\sqrt{2}}{2}$

Áp dụng BĐT $AM-GM$ ta có:

$\sum \sqrt{\frac{a^2+1}{b^2+6bc+c^2}}=\sum \sqrt{\frac{a^2+ab+bc+ca}{(b+c)^2+4bc}}$

 

$=\sum \sqrt{\frac{(a+b)(c+a)}{(b+c)^2+4bc}}$

 

$ \geq \sum \sqrt{\frac{(a+b)(c+a)}{2(b+c)^2}}$

 

$\geq 3.\sqrt[6]{\frac{(a+b)^2(b+c)^2(c+a)^2}{8(a+b^2)(b+c)^2(c+a)^2}}=\frac{3\sqrt{2}}{2}$ 

Xảy ra dấu $=$ khi $a=b=c=\frac{1}{\sqrt{3}}$


Edited by khanghaxuan, 18-06-2015 - 18:32.


#302
dangkhuong

dangkhuong

    Sĩ quan

  • Thành viên
  • 312 posts

Bài 112:(Canada MO): Cho a,b,c>0 sao cho a+b+c=3. Cmr: 

 

$\frac{a}{\sqrt{a^3+a^2+1}}+\frac{b}{\sqrt{b^3+b^2+1}}+\frac{c}{\sqrt{c^3+c^2+1}}\leq \sqrt{3}$


Edited by khanghaxuan, 18-06-2015 - 18:35.

:ukliam2:  :ukliam2:  :ukliam2:


#303
dangkhuong

dangkhuong

    Sĩ quan

  • Thành viên
  • 312 posts

Bài 113:(Palestine MO): Cho a,b,c>0. Cmr:

 

$\frac{(a^3+2abc)^3}{(2a^3+c^3)^2}+\frac{(b^3+2abc)^3}{(2b^3+c^3)^2}+\frac{(c^3+2abc)^3}{(2c^3+a^3)^2}\leq 3(a^3+b^3+c^3)$


Edited by khanghaxuan, 18-06-2015 - 18:33.

:ukliam2:  :ukliam2:  :ukliam2:


#304
Hoang Nhat Tuan

Hoang Nhat Tuan

    Hỏa Long

  • Thành viên
  • 974 posts

Bài 112:(Canada MO)(Nguyễn Duy Khương): Cho a,b,c>0 sao cho a+b+c=3. Cmr: :ukliam2:  :ukliam2:  :ukliam2: 

 

$\frac{a}{\sqrt{a^3+a^2+1}}+\frac{b}{\sqrt{b^3+b^2+1}}+\frac{c}{\sqrt{c^3+c^2+1}}\leq \frac{\sqrt{3}}{3}$

Đề sai rồi, với $a=b=c=1$ thì VT>VT

VP phải là $\sqrt{3}$ chứ :(


Ngài có thể trói cơ thể tôi, buộc tay tôi, điều khiển hành động của tôi: ngài mạnh nhất, và xã hội cho ngài thêm quyền lực; nhưng với ý chí của tôi, thưa ngài, ngài không thể làm gì được.

#305
nangcuong8e

nangcuong8e

    Trung sĩ

  • Thành viên
  • 134 posts

Bài 112:(Canada MO): Cho a,b,c>0 sao cho a+b+c=3. Cmr: 

 

$\frac{a}{\sqrt{a^3+a^2+1}}+\frac{b}{\sqrt{b^3+b^2+1}}+\frac{c}{\sqrt{c^3+c^2+1}}\leq \sqrt{3}$

Ta có: $\sum \frac{a}{\sqrt{a^3 +a^2+1}} \leq \sqrt{3(\sum \frac{a^2}{a^3 +a^2 +1})}$ 
 Do đó ta cần chứng minh $\sum \frac{a^2}{a^3 +a^2+1} \leq 1$

Hay cụ thể ta phải chứng minh $\frac{a^2}{a^3+a^2+1} \leq \frac{a+2}{9}$ (1)

 Thật vậy, (1) $\Leftrightarrow a^4 +3a^3 -7a^2 +a +2 \geq 0$

$\Leftrightarrow (a-1)^2(a^2+5a+2) \geq 0$ (luôn đúng với $a >0$)

$\Rightarrow \sum \frac{a^2}{a^3 +a^2 +1} \leq \sum \frac{a+2}{9} = \frac{3 +2.3}{9} =1$

 Dấu $"="$ xảy ra khi $a=b=c=1$


Edited by nangcuong8e, 18-06-2015 - 19:14.


#306
Belphegor Varia

Belphegor Varia

    Thượng sĩ

  • Thành viên
  • 227 posts

Bài 114 (VMO 1999) : Xét các số thực dương $a,b$ sao cho phương trình : 

                                                                                     $ax^{3}-x^{2}+bx-1=0$

có ba nghiệm thực dương ( các nghiệm có thể bằng nhau).

 

Tìm giá trị nhỏ nhất của biểu thức $P=\frac{5a^{2}-3ab+2}{a^{2}(b-a)}$ .


$ \textbf{NMQ}$

Wait a minute, You have enough time. Also tomorrow will come 

Just take off her or give me a ride 

Give me one day or one hour or just one minute for a short word 

 


#307
Hoang Tung 126

Hoang Tung 126

    Thiếu tá

  • Thành viên
  • 2061 posts

Bài 114 (VMO 1999) : Xét các số thực dương $a,b$ sao cho phương trình : 

                                                                                     $ax^{3}-x^{2}+bx-1=0$

có ba nghiệm thực dương ( các nghiệm có thể bằng nhau).

 

Tìm giá trị nhỏ nhất của biểu thức $P=\frac{5a^{2}-3ab+2}{a^{2}(b-a)}$ .

 Gọi $x_{1},x_{2},x_{3}$ là 3 nghiệm dương của phương trình. 

   Theo định lý Viet cho phương trình bậc 3 ta có :

 

  $\left\{\begin{matrix} x_{1}x_{2}+x_{2}x_{3}+x_{3}x_{1}=\frac{b}{a} & & \\ x_{1}+x_{2}+x_{3}=\frac{1}{a} & & \\ x_{1}x_{2}x_{3}=\frac{1}{a} & & \end{matrix}\right.$

 

Ta có :$\frac{b}{a^2}=\frac{b}{a}.\frac{1}{a}=(\sum x_{1}x_{2})(\sum x_{1})$

           $\frac{\sum x_{1}x_{2}}{x_{1}x_{2}x_{3}}=\frac{\frac{b}{a}}{\frac{1}{a}}=b= > \sum \frac{1}{x_{1}}=b= > \frac{b}{a}=(\sum \frac{1}{x_{1}})(\sum x_{1})$

( Do $\sum x_{1}=\frac{1}{a}$)

Theo Cosi ta có :$\frac{1}{a}=x_{1}x_{2}x_{3}=\sum x_{1}\geq 3\sqrt[3]{x_{1}x_{2}x_{3}}= > (x_{1}x_{2}x_{3})^3\geq 27x_{1}x_{2}x_{3}= > x_{1}x_{2}x_{3}\geq \sqrt{27}=3\sqrt{3}$

 

  Từ đó 

$P=\frac{5a^2-3ab+2}{a^2(b-a)}$

$=\frac{\frac{5a^2-3ab+2}{a^3}}{\frac{a^2(b-a)}{a^3}}=\frac{\frac{5}{a}-\frac{3b}{a^2}+\frac{2}{a^3}}{\frac{b}{a}-1}$

$=\frac{5x_{1}x_{2}x_{3}-3(\sum x_{1})(\sum x_{1}x_{2})+2(\sum x_{1})^3}{(\sum x_{1})(\sum \frac{1}{x_{1}})-1}$

$=(x_{1}x_{2}x_{3})(\frac{5x_{1}x_{2}x_{3}-3(\sum x_{1})(\sum x_{1}x_{2})+2(\sum x_{1})^3}{(\sum x_{1})(\sum x_{1}x_{2})-x_{1}x_{2}x_{3}})$

$\geq 3\sqrt{3}.(\frac{5x_{1}x_{2}x_{3}-3\sum x_{1}x_{2}(x_{1}+x_{2})-9x_{1}x_{2}x_{3}+2(\sum x_{1})^3}{\sum x_{1}x_{2}(x_{1}+x_{2})+2x_{1}x_{2}x_{3}})$

$= > P\geq 3\sqrt{3}.\left [ \frac{-4x_{1}x_{2}x_{3}-3\sum x_{1}x_{2}(x_{1}+x_{2})+2(\sum x_{1})^3}{\sum x_{1}x_{2}(x_{1}+x_{2})+2x_{1}x_{2}x_{3}} \right ]$

 

Ta sẽ chứng minh:   $P\geq 12\sqrt{3}$

$< = > \frac{-4x_{1}x_{2}x_{3}-3\sum x_{1}x_{2}(x_{1}+x_{2})+2(\sum x_{1})^3}{\sum x_{1}x_{2}({x_{1}+x_{2}})+2x_{1}x_{2}x_{3}}\geq 4$

$< = > 2(\sum x_{1})^3-3\sum x_{1}x_{2}(x_{1}+x_{2})-4x_{1}x_{2}x_{3}\geq 4\sum x_{1}x_{2}(x_{1}+x_{2})+8x_{1}x_{2}x_{3}$

$< = > 2\sum x_{1}^3+6\sum x_{1}x_{2}(x_{1}+x_{2})+12x_{1}x_{2}x_{3}-3\sum x_{1}x_{2}(x_{1}+x_{2})-4x_{1}x_{2}x_{3}$

$\geq 4\sum x_{1}x_{2}(x_{1}+x_{2})+8x_{1}x_{2}x_{3}$

$< = > 2\sum x_{1}^3\geq \sum x_{1}x_{2}(x_{1}+x_{2})$

$< = > \sum (x_{1}+x_{2})(x_{1}-x_{2})^2\geq 0$

 ( Điều này luôn đúng với $x_{1},x_{2},x_{3}> 0$)

 

  Do đó $P\geq 12\sqrt{3}= > P_{min}=12\sqrt{3}< = > x_{1}=x_{2}=x_{3}< = > a=\frac{1}{3\sqrt{3}},b=\sqrt{3}$


Edited by Hoang Tung 126, 18-06-2015 - 22:22.


#308
Belphegor Varia

Belphegor Varia

    Thượng sĩ

  • Thành viên
  • 227 posts

Bài 115 (CĐTMO 2006) : Chứng minh rằng với mọi số thực $x,y,z$ thuộc đoạn $[1;2]$ ta luôn có bất đẳng thức :
                                          $(x+y+z)(\frac{1}{x}+\frac{1}{y}+\frac{1}{z})\geq 6(\frac{x}{y+z}+\frac{y}{x+z}+\frac{z}{x+y}).$

                                           Hỏi đẳng thức xảy ra khi và chỉ khi nào ? 

Bài 116 (CĐTMO 2005) : Chứng minh rằng
                                                                 $\frac{a^{3}}{(b+c)^{3}}+\frac{b^{3}}{(c+a)^{3}}+\frac{c^{3}}{(a+b)^{3}}$ $\geq \frac{3}{8}$

                                        trong đó $a,b,c$ là các số dương.

Bài 117 (CĐTMO 2001) : Xét các số thực dương $a,b,c$ thỏa mãn điều kiện 

                                                                          $21ab+2bc+8ca\leq 12$
                                        Tìm giá trị nhỏ nhất của biểu thức $P=\frac{1}{a}+\frac{2}{b}+\frac{3}{c}.$ 

 

Spoiler


 


Edited by Belphegor Varia, 19-06-2015 - 14:36.

$ \textbf{NMQ}$

Wait a minute, You have enough time. Also tomorrow will come 

Just take off her or give me a ride 

Give me one day or one hour or just one minute for a short word 

 


#309
Nguyen Minh Hai

Nguyen Minh Hai

    Thiếu úy

  • Thành viên
  • 666 posts

 

Bài 117 (CĐTMO 2001) : Xét các số thực dương $a,b,c$ thỏa mãn điều kiện 

                                                                          $21ab+2bc+8ca\leq 12$
                                        Tìm giá trị nhỏ nhất của biểu thức $P=\frac{1}{a}+\frac{2}{b}+\frac{3}{c}.$ 

 

Spoiler


 

Bài toán này đã có khá nhiều cách giải trên diễn đàn

http://diendantoanho...-frac2b-frac3c/

http://diendantoanho...c3cgeq-frac152/

Thêm một cách mà mình vừa nghĩ ra  :icon10: 

Từ điểm rơi của bài toán là $(a,b,c)=\left ( \frac{1}{3};\frac{4}{5};\frac{3}{2} \right )$

$\Rightarrow 36a=15b=8c=12$

Do đó ta đặt: $\left\{\begin{matrix} 36a=x & & & \\ 15b=y & & & \\ 8c=z & & & \end{matrix}\right.$

Bài toán trở thành: 

Cho $x,y,z>0$ thõa mãn: $\frac{7xy}{180}+\frac{yz}{60}+\frac{zx}{36} \leq 12$.TÌm GTNN của:

$P=\frac{36}{x}+\frac{30}{y}+\frac{24}{z}$

Từ giả thiết ta có: 

$2160 \geq 7xy+3yz+5zx \geq 15\sqrt[15]{x^7y^7.y^3z^3.z^5x^5}$

$\Rightarrow x^6y^5z^4 \leq 12^5$            (Theo $AM-GM$)

Do đó: $P=6(\frac{6}{x}+\frac{5}{y}+\frac{4}{z})\geq 6.15\sqrt[15]{\frac{1}{x^6y^5z^4}} \geq 90.\sqrt[15]{\frac{1}{12^{15}}}=\frac{15}{2}$

$\Rightarrow GTNN_{P}=\frac{15}{2}$ khi $(a,b,c)=\left ( \frac{1}{3};\frac{4}{5};\frac{3}{2} \right )$

[spoiler] Đề thi vòng 16 Vio 9 năm nay cũng ra bài này [\spoiler]


Edited by Nguyen Minh Hai, 19-06-2015 - 09:47.


#310
Nguyen Minh Hai

Nguyen Minh Hai

    Thiếu úy

  • Thành viên
  • 666 posts

 

Bài 116 (CĐTMO 2005) : Chứng minh rằng
                                                                 $\frac{a^{3}}{(b+c)^{3}}+\frac{b^{3}}{(c+a)^{3}}+\frac{c^{3}}{(a+b)^{3}}$

                                        trong đó $a,b,c$ là các số dương.

 

 

Có vẽ đề bị thiếu thì phải?  :closedeyes:



#311
dogsteven

dogsteven

    Đại úy

  • Thành viên
  • 1567 posts

Bài toán này đã có khá nhiều cách giải trên diễn đàn

http://diendantoanho...-frac2b-frac3c/

http://diendantoanho...c3cgeq-frac152/

Thêm một cách mà mình vừa nghĩ ra  :icon10: 

Từ điểm rơi của bài toán là $(a,b,c)=\left ( \frac{1}{3};\frac{4}{5};\frac{3}{2} \right )$

$\Rightarrow 36a=15b=8c=12$

Do đó ta đặt: $\left\{\begin{matrix} 36a=x & & & \\ 15b=y & & & \\ 8c=z & & & \end{matrix}\right.$

Bài toán trở thành: 

Cho $x,y,z>0$ thõa mãn: $\frac{7xy}{180}+\frac{yz}{60}+\frac{zx}{36} \leq 12$.TÌm GTNN của:

$P=\frac{36}{x}+\frac{30}{y}+\frac{24}{z}$

Từ giả thiết ta có: 

$2160 \geq 7xy+3yz+5zx \geq 15\sqrt[15]{x^7y^7.y^3z^3.z^5x^5}$

$\Rightarrow x^6y^5z^4 \leq 12^5$            (Theo $AM-GM$)

Do đó: $P=6(\frac{6}{x}+\frac{5}{y}+\frac{4}{z})\geq 6.15\sqrt[15]{\frac{1}{x^6y^5z^4}} \geq 90.\sqrt[15]{\frac{1}{12^{15}}}=\frac{15}{2}$

$\Rightarrow GTNN_{P}=\frac{15}{2}$ khi $(a,b,c)=\left ( \frac{1}{3};\frac{4}{5};\frac{3}{2} \right )$

[spoiler] Đề thi vòng 16 Vio 9 năm nay cũng ra bài này [\spoiler]

Lời giải không quan trọng, tìm được điểm rơi mới là điều tất yếu để giải quyết bài toán.

Không biết có cái hệ nào gọn hơn không chứ AM-GM suy rộng và Lagrange đều ra cái hệ như thế này:

$6yz-1=6zx-4=4xy-7$ và $2x+8y+21z=12xyz$ với $ax=by=cz=1$

Tuy nhiên, vẫn thích cách thế trực tiếp hơn, vừa tự nhiên, vừa đỡ tốn sức tìm điểm rơi.


Edited by dogsteven, 19-06-2015 - 12:35.

Quyết tâm off dài dài cày hình, số, tổ, rời rạc.


#312
dogsteven

dogsteven

    Đại úy

  • Thành viên
  • 1567 posts

Có vẽ đề bị thiếu thì phải?  :closedeyes:

Chắc là chứng minh nó lớn hơn hoặc bằng $\dfrac{3}{8}$ :))


Quyết tâm off dài dài cày hình, số, tổ, rời rạc.


#313
dogsteven

dogsteven

    Đại úy

  • Thành viên
  • 1567 posts

Bài 115. 

Bất đẳng thức đã cho tương đương với: $\sum \left[\dfrac{1}{yz}-\dfrac{3}{(x+y)(x+z)}\right](y-z)^2\geqslant 0$

(1) $(y+z)(y+x)-3zx=y^2+y(z+x)-2zx=y(y+z)+xy-2zx\geqslant 2x(y-z)\geqslant 0$

(2) $\dfrac{1}{yz}-\dfrac{3}{(x+y)(x+z)}\geqslant \dfrac{1}{4yz}\geqslant 0$

(3) $\dfrac{1}{zx}+\dfrac{1}{xy}-\dfrac{3}{(y+z)(y+x)}-\dfrac{3}{(z+x)(z+y)}=\dfrac{x^2(y^2+z^2)+x(y^3+z^3)+yz(y+z)^2-4x^2yz}{xyz(x+y)(y+z)(z+x)}$

Mà $x^2(y^2+z^2)+x(y^3+z^3)+yz(y+z)^2-4x^2yz\geqslant x(y^3+z^3)-x^2yz\geqslant yz(y+z-x)\geqslant 0$

nên $\dfrac{1}{zx}+\dfrac{1}{xy}-\dfrac{3}{(y+z)(y+x)}-\dfrac{3}{(z+x)(z+y)}\geqslant 0$

Theo tiêu chuẩn S.O.S ta có điều phải chứng minh.


Edited by dogsteven, 19-06-2015 - 13:01.

Quyết tâm off dài dài cày hình, số, tổ, rời rạc.


#314
Nguyenhuyen_AG

Nguyenhuyen_AG

    Trung úy

  • Thành viên nổi bật 2016
  • 945 posts
Bài 116 (CĐTMO 2005) : Chứng minh rằng

                                                                 $\frac{a^{3}}{(b+c)^{3}}+\frac{b^{3}}{(c+a)^{3}}+\frac{c^{3}}{(a+b)^{3}}$

                                        trong đó $a,b,c$ là các số dương.

 

Đề bài này sai rồi, đề đúng là như vầy

\[\frac{a^{3}}{(a+b)^{3}}+\frac{b^{3}}{(b+c)^{3}}+\frac{c^{3}}{(c+a)^{3}} \geqslant \frac{3}{8}.\]

Tác giả là thầy Nam Dũng.

 

Lời giải 1. Sử dụng bất đẳng thức Cauchy-Schwarz, ta có \[\left [ \sum \left ( \frac{a}{a+b} \right )^3 \right ]\left [ \sum c^3(a+b)^3 \right ] \ge \left (\sum c^{\frac{3}{2}}a^{\frac{3}{2}} \right )^2,\] như vậy ta cần chứng minh được \[8\left (\sum c^{\frac{3}{2}}a^{\frac{3}{2}}\right )^2 \ge 3 \sum c^3(a+b)^3 .\] Đặt $x=\sqrt{ab},\,y=\sqrt{bc},\,z=\sqrt{ca},$ bất đẳng thức trên trở thành \[8\left (\sum z^3\right )^2 \ge 3 \sum (y^2+z^2)^3,\] hay là \[\sum (x^6+y^6)+16\sum x^3y^3 \ge 9 \sum x^2y^2(x^2+y^2),\] bất đẳng thức này đúng vì \[x^6+y^6+16x^3y^3-9x^2y^2(x^2+y^2) =(x-y)^4(x^2+4xy+y^2) \ge 0.\] Chứng minh hoàn tất.

 

Lời giải 2. Theo bất đẳng thức trung bình lũy thừa, ta có \[\sqrt[3]{\frac{\displaystyle\sum \left ( \frac{a}{a+b} \right )^3}{3}}\ge\sqrt{\frac{\displaystyle\sum \left ( \frac{a}{a+b} \right )^2}{3}}.\] Như vậy, để hoàn tất chứng minh thì ta cần chỉ ra được \[\left ( \frac{a}{a+b} \right )^2+\left ( \frac{b}{b+c} \right )^2+\left ( \frac{c}{c+a} \right )^2 \ge \frac{3}{4}.\] Thay $\left ( \frac{b}{a},\,\frac{c}{b},\,\frac{a}{c} \right )$ bởi $(x,\,y,\,z)$ ta sẽ đưa bài toán về chứng minh \[\frac{1}{(x+1)^2}+\frac{1}{(y+1)^2}+\frac{1}{(z+1)^2}\ge\frac{3}{4},\] với điều kiện $xyz=1.$

 

Sử dụng bất đẳng thức Cauchy-Schwarz, ta có \[\frac{1}{(x+1)^2}+\frac{1}{(y+1)^2}\ge \frac{1}{(\frac{x}{y}+1)(xy+1)}+\frac{1}{(\frac{y}{x}+1)(xy+1)}=\frac{1}{xy+1}.\] Ta sẽ chứng minh \[\frac{1}{xy+1}+\frac{1}{(z+1)^2}\ge \frac{3}{4},\] hay là \[\frac{z}{1+z}+\frac{1}{(z+1)^2}\ge \frac{3}{4}.\] Bất đẳng thức này đúng bởi vì \[\frac{z}{1+z}+\frac{1}{(z+1)^2}-\frac{3}{4}=\frac{(z-1)^2}{4(z+1)} \ge 0.\]
Nhận xét. Một cách khác để chứng minh bất đẳng thức \[\left ( \frac{a}{a+b} \right )^2+\left ( \frac{b}{b+c} \right )^2+\left ( \frac{c}{c+a} \right )^2 \ge \frac{3}{4}.\] Sử dụng bất đẳng thức Cauchy-Schwarz, ta có \[\left [ \sum \left ( \frac{a}{a+b} \right )^2 \right ]\left [ \sum (a+b)^2(a+c)^2 \right ]\ge \left [ \sum a(a+c) \right ]^2.\] Tuy nhiên \[4\left [ \sum a(a+c) \right ]^2 = \left [ \sum (a+b)^2 \right ]^2\] Vậy, ta chỉ cần chứng minh \[\left [ \sum (a+b)^2 \right ]^2 \ge 3 \sum (a+b)^2(a+c)^2,\] nhưng bất đẳng thức này đúng theo bất đẳng thức cơ bản \[(x+y+z)^2\ge 3(xy+yz+zx).\] Bất đẳng thức  được chứng minh.

 

Ngoài ra ta có thể chứng minh bài toán bằng cách sử dụng bất đẳng thức China TST 2004 của giáo sư Vasile Cirtoaje

\[\frac{a^2}{(a+b)^2}+\frac{b^2}{(b+c)^2}+\frac{c^2}{(c+d)^2}+\frac{d^2}{(d+a)^2} \geqslant 1.\]

 

 

Bài 115 (CĐTMO 2006) : Chứng minh rằng với mọi số thực $x,y,z$ thuộc đoạn $[1;2]$ ta luôn có bất đẳng thức :
                                          $(x+y+z)(\frac{1}{x}+\frac{1}{y}+\frac{1}{z})\geq 6(\frac{x}{y+z}+\frac{y}{x+z}+\frac{z}{x+y}).$

                                           Hỏi đẳng thức xảy ra khi và chỉ khi nào ?
 

 

Đặt \[f(x,\,y,\,z)=(x+y+z)\left( \frac{1}{x}+\frac{1}{y}+\frac{1}{z} \right)-6\left( \frac{x}{y+z}+\frac{y}{z+x}+\frac{z}{x+y} \right),\]

ta sẽ chứng minh $f(x,\,y,\,z)\ge 0$ bằng kỹ thuật dồn biến. Thật vậy, ta có $f(x,\,y,\,z)-f\left ( x,\,\frac{y+z}{2},\, \frac{y+z}{2}\right ),$ sẽ bằng \[\frac{(x+y+z)(y-z)^2}{yz(y+z)}-\frac{6(x+y+z)(y-z)^2}{(x+y)(x+z)(2x+y+z)},\] hay \[\frac{(x+y+z)(y-z)^2[(x+y)(z+x)(2x+y+z)-6yz(y+z)]}{(x+y)(y+z)(z+x)(2x+y+z)}.\] Giả sử $x$ là số lớn nhất trong ba số thì $(x+y)(z+x)(2x+y+z) > 6yz(y+z),$ nên $f(x,\,y,\,z)\ge f(x,\,t,\,t)$ với $t=\frac{y+z}{2} \ge 1.$

 

Tiếp đến, ta sẽ chứng minh $f(x,\,t,\,t)\ge 0.$ Bất đẳng thức tương đương với

$$(x+2t)\left( \frac{1}{x}+\frac{2}{t} \right)-6\left( \frac{x}{2t}+\frac{2t}{t+x} \right)\ge 0,$$

$$\frac{{{(t-x)}^{2}}(2t-x)}{tx(t+x)}\ge 0,$$

hiển nhiên đúng vì $2t\ge 2\ge x.$ Đẳng thức xảy ra khi và chỉ khi $x=y=z,$ hoặc $x=2$ và $y=z=1.$ Bài toán được chứng minh.

 

Nhận xét. Nếu $x,\,y,\,z$ là độ dài ba cạnh của tam giác thì bài toán vẫn đúng. Thật vậy, vì $x,\,y,\,z$ là độ dài ba cạnh của tam giác nên, theo phép thế Ravi, ta có thể viết bất đẳng thức lại như sau \[(x+y+z)\sum \frac{1}{x+y} \ge 3\sum \frac{y+z}{2x+y+z},\] tương đương với \[2\sum x^2(x^2-y^2)(x^2-z^2)+3\sum yz(y^2-z^2)^2 \ge 0,\] hiển nhiên đúng theo bất đẳng thức Shur.

 

Tác giả bài toán cũng là thầy Nam Dũng.


Edited by Nguyenhuyen_AG, 19-06-2015 - 13:24.

Nguyen Van Huyen
Ho Chi Minh City University Of Transport

#315
Hoang Tung 126

Hoang Tung 126

    Thiếu tá

  • Thành viên
  • 2061 posts

Bài 115 (CĐTMO 2006) : Chứng minh rằng với mọi số thực $x,y,z$ thuộc đoạn $[1;2]$ ta luôn có bất đẳng thức :
                                          $(x+y+z)(\frac{1}{x}+\frac{1}{y}+\frac{1}{z})\geq 6(\frac{x}{y+z}+\frac{y}{x+z}+\frac{z}{x+y}).$

                                           Hỏi đẳng thức xảy ra khi và chỉ khi nào ? 

 

  Không mất tổng quát giả sử $x\geq y\geq z$ .Do $2\geq x,y,z\geq 1= > \left\{\begin{matrix} x+y\geq z & & \\ y+z\geq x & & \\ z+x\geq y & & \end{matrix}\right.$

 

BĐT $< = > (\sum x)(\sum \frac{1}{x})\geq 6(\sum \frac{x}{y+z})< = > (\sum x)(\sum \frac{1}{x})-9\geq 6(\sum \frac{x}{y+z}-\frac{3}{2})$

 

 Ta có :$(\sum x)(\sum \frac{1}{x})-9=\sum \frac{(y-z)^2}{yz}$

            $\sum \frac{x}{y+z}-\frac{3}{2}=\sum \frac{(y-z)^2}{2(y+x)(z+x)}$

 

 Do đó BĐT $< = > \sum \frac{(y-z)^2}{yz}\geq 6(\sum \frac{(y-z)^2}{2(y+x)(z+x)})$

$< = > \sum \frac{(y-z)^2}{yz}\geq 3\sum \frac{(y-z)^2}{(y+x)(z+x)}< = > \sum (y-z)^2(\frac{1}{yz}-\frac{3}{(y+x)(z+x)})\geq 0$

$< = > (x-y)^2(\frac{1}{xy}-\frac{3}{(x+z)(y+z)})+(y-z)^2(\frac{1}{yz}-\frac{3}{(y+x)(z+x)})+(x-z)^2(\frac{1}{xz}-\frac{3}{(x+y)(z+y)})\geq 0$

 

  Đặt $\left\{\begin{matrix} S_{a}=\frac{1}{yz}-\frac{3}{(y+x)(z+x)} & & \\ S_{b}=\frac{1}{xz}-\frac{3}{(x+y)(z+y)} & & \\ S_{c}=\frac{1}{xy}-\frac{3}{(x+z)(y+z)} & & \end{matrix}\right.$

 

Ta có :$S_{b}=\frac{1}{xz}-\frac{3}{(x+y)(z+y)}=\frac{xy+yz+y^2-2xz}{xz(x+y)(z+y)}=\frac{x(y-z)+y(y+z)-xz}{xz(x+y)(z+y)}\geq \frac{x(y-z)+xy-xz}{xz(x+y)(z+y)}=\frac{2x(y-z)}{xz(x+y)(z+y)}=\frac{2(y-z)}{z(x+y)(z+y)}\geq 0= > S_{b}\geq 0$

  (Do $y+z\geq x= > y(y+z)\geq xy,y\geq z= > y-z\geq 0$)

          $S_{a}=\frac{1}{yz}-\frac{3}{(x+y)(x+z)}=\frac{x^2+xy+xz-2yz}{yz(x+y)(x+z)}=\frac{x^2+y(x-z)+z(x-y)}{yz(x+y)(x+z)}> 0= > S_{a}> 0$ (Do $x\geq z= > y(x-z)\geq 0,x\geq y= > z(x-y)\geq 0,x^2> 0= > x^2+y(x-z)+z(x-y)> 0$)

  Do đó $S_{a}+S_{b}> 0$

 

Ta có :$S_{b}+S_{c}=\frac{1}{xz}+\frac{1}{xy}-\frac{3}{(x+y)(z+y)}-\frac{3}{(x+z)(y+z)}$

$=\frac{y+z}{xyz}-\frac{3(x+z)+3(x+y)}{(x+y)(y+z)(x+z)}=\frac{y+z}{xyz}-\frac{6x+3y+3z}{(x+y)(y+z)(x+z)}$

  Ta chứng minh $S_{b}+S_{c}\geq 0< = > \frac{y+z}{xyz}\geq \frac{6x+3y+3z}{(x+y)(y+z)(x+z)}< = > (x+y)(x+z)(y+z)^2\geq xyz(6x+3y+3z)< = > (x^2+xz+xy+yz)(y^2+2yz+z^2)\geq 6x^2yz+3xy^2z+3xyz^2< = > x^2y^2+2x^2yz+x^2z^2+xy^2z+2xyz^2+xz^3+xy^3+2xy^2z+xyz^2+y^3z+2y^2z^2+yz^3\geq 6x^2yz+3xy^2z+3xyz^2< = > x^2y^2+x^2z^2+y^3z+yz^3+2y^2z^2+xz^3+xy^3\geq 4x^2yz$

               $< = > yz(y+z)^2+x^2(y^2+z^2)+x(y^3+z^3)\geq 4x^2yz$

  BĐT này đúng do theo Cosi ta có :

   $x^2(y^2+z^2)\geq x^2.2yz=2x^2yz$

   $x(y^3+z^3)\geq x.yz(y+z)\geq x^2yz$ ( Do $y+z\geq x$)

   $yz(y+z)^2\geq x^2yz$ (Do $y+z\geq x$

 

Cộng theo vế $= > yz(y+z)^2+x^2(y^2+z^2)+x(y^3+z^3)\geq 4x^2yz$ (Luôn đúng)

 

   Do đó $S_{b}+S_{c}\geq 0$

 

Từ đó $S_{b}\geq 0,S_{b}+S_{a}> 0,S_{b}+S_{c}\geq 0$ nên theo nguyên lý SOS thì BĐT đề bài được chứng minh. 

 

    Dấu = xảy ra khi $x=y=z$ hoặc $x=2y=2z$ và hoán vị của chúng


Edited by Hoang Tung 126, 19-06-2015 - 13:21.


#316
dangkhuong

dangkhuong

    Sĩ quan

  • Thành viên
  • 312 posts

Bài 113:(Palestine MO): Cho a,b,c>0. Cmr:

 

$\frac{(a^3+2abc)^3}{(2a^3+c^3)^2}+\frac{(b^3+2abc)^3}{(2b^3+c^3)^2}+\frac{(c^3+2abc)^3}{(2c^3+a^3)^2}\leq 3(a^3+b^3+c^3)$

sau đây là lời giải cho bài 113:(vắn tắt như sau)

 

1) theo bất đẳng thức Holder thì $(a^3+b^3+b^3)(a^3+a^3+c^3)(a^3+a^3+c^3)\geq (a^3+bc^2+ba^2)^3=(a^3+b(a^2+c^2))^3\geq (a^3+2abc)^3$

 Do đó ta có $\frac{(a^3+2abc)^3}{(2a^3+c^3)^2}\leq a^3+2b^3$

2)Tương tự ta có các bất đẳng thức sau:

 $\frac{(b^3+2abc)^3}{(2b^3+c^3)^2}\leq b^3+2c^3$

$\frac{(c^3+2abc)^3}{(2c^3+a^3)^2}\leq c^3+2a^3$

 Cộng ba vế bất đẳng thức trên ta có đpcm.(lời giải của tác giả thì mình ko bit đây là lời giải của mình)


Edited by dangkhuong, 19-06-2015 - 15:51.

:ukliam2:  :ukliam2:  :ukliam2:


#317
dangkhuong

dangkhuong

    Sĩ quan

  • Thành viên
  • 312 posts

Bài 118(Sweeden MO): Cho a,b,c>0. Cmr nếu a+b+c=3 thì 

      $\sqrt[5]{\frac{a^3+b^3+1}{b^2+2c^2}}+\sqrt[5]{\frac{b^3+c^3+1}{c^2+2a^2}}+\sqrt[5]{\frac{c^3+a^3+1}{a^2+2b^2}}\geq \frac{9abc}{ab+bc+ca}$


Edited by dangkhuong, 19-06-2015 - 16:06.

:ukliam2:  :ukliam2:  :ukliam2:


#318
dangkhuong

dangkhuong

    Sĩ quan

  • Thành viên
  • 312 posts

Bài 119(Thailand MO): Cho a,b,c>0. CMR: 

$\frac{a^2}{b^2+bc+c^2}+\frac{b^2}{c^2+ca+a^2}+\frac{c^2}{a^2+ab+b^2}\geq 1$

 


Edited by dangkhuong, 19-06-2015 - 16:09.

:ukliam2:  :ukliam2:  :ukliam2:


#319
Belphegor Varia

Belphegor Varia

    Thượng sĩ

  • Thành viên
  • 227 posts

Bài 119(Thailand MO): Cho a,b,c>0. CMR: :ukliam2:  :ukliam2:  :ukliam2: 

$\frac{a^2}{b^2+bc+c^2}+\frac{b^2}{c^2+ca+a^2}+\frac{c^2}{a^2+ab+b^2}\geq 1$

 

(bài này cực hay....)

Spam : Mình xin góp ý là bạn chỉ nên post đề bài để mọi người cùng thảo luận là đủ  ^_^ , không nên để thêm mấy icon hoặc lời nhắn không cần thiết cạnh đề bài


$ \textbf{NMQ}$

Wait a minute, You have enough time. Also tomorrow will come 

Just take off her or give me a ride 

Give me one day or one hour or just one minute for a short word 

 


#320
dangkhuong

dangkhuong

    Sĩ quan

  • Thành viên
  • 312 posts

Xin trình bày lời giải bài số 119 như sau:

 

Không mất tính tổng quát ta giả sử rằng $a\geq b\geq c$

 

khi đó ta dễ thấy rằng: $a^2\geq b^2\geq c^2$ ; $\frac{1}{b^2+c^2+bc}\geq \frac{1}{c^2+ca+a^2}\geq \frac{1}{a^2+ab+b^2}$

 

Áp dụng bất đẳng thức Chebysev,và bất đẳng thức AM-GM ta có:

$\frac{a^2}{b^2+bc+c^2}+\frac{b^2}{c^2+ca+a^2}+\frac{c^2}{a^2+ab+b^2}\geq \frac{1}{3}(a^2+b^2+c^2)(\sum \frac{1}{b^2+bc+c^2})\geq 3(a^2+b^2+c^2)(\frac{1}{2(a^2+b^2+c^2)+ab+bc+ca})\geq 3(a^2+b^2+c^2)(\frac{1}{3(a^2+b^2+c^2)})=1$

 

Vậy là ta có đpcm. :ukliam2:  :ukliam2:  :ukliam2: 


:ukliam2:  :ukliam2:  :ukliam2:





1 user(s) are reading this topic

0 members, 1 guests, 0 anonymous users